LSAT and Law School Admissions Forum

Get expert LSAT preparation and law school admissions advice from PowerScore Test Preparation.

 Administrator
PowerScore Staff
  • PowerScore Staff
  • Posts: 8926
  • Joined: Feb 02, 2011
|
#31772
Please post below with any questions!
 brcibake
  • Posts: 55
  • Joined: Jul 19, 2017
|
#38649
What is the reasoning behind this answer. I thought it was B and I am not sure were I went wrong.
 nicholaspavic
PowerScore Staff
  • PowerScore Staff
  • Posts: 271
  • Joined: Jun 12, 2017
|
#39106
Hi brci,

The correct answer is Answer (B):

"Jury decisions in cases involving expert witness testimonies are not always determined by the reliability of those testimonies."

This answer choice for this Must Be True question passes the Fact Test and it therefore the right answer. I hope this was good news!
:-D
User avatar
 emilyjmyer
  • Posts: 48
  • Joined: May 11, 2022
|
#97559
Hi!

Why is the answer B and not C?

I realized that the words "not always" made for an attractive MBT answer choice but I still went with C.

I chose C because in the stimulus it describes

experts give testimony --> jurors don't understand technical info --> can't evaluate info

I thought C was a contrapositive: Jurors can evaluate info--> jurors do understand technical info

Where did I go wrong here?

Thanks!
 Robert Carroll
PowerScore Staff
  • PowerScore Staff
  • Posts: 1787
  • Joined: Dec 06, 2013
|
#97677
emilyjmyer,

Answer choice (C) doesn't talk about understanding the information. It talks about assessing the legal implications, which is not discussed in the stimulus.

Additionally, it looks like what you diagramming shows answer choice (C) to be closer to a Mistaken Negation than anything else. Your diagram of the stimulus has "not understanding" and "not evaluate" as separate conditions. So in answer choice (C), the sufficient condition would be "understand" anyway. Your diagram of answer choice (C) doesn't match the conditionality of it anyway. Hope this helps!

Robert Carroll
User avatar
 willwants170
  • Posts: 10
  • Joined: Dec 05, 2023
|
#106543
Would B be correct because it was most supported, not fully supported? When reading B and relating it to the stimulus, wouldn't it be possible that jury decisions are always informed by reliability of testimonies even if the jurors are unable to assess the reliability. I ended up not picking B because I thought, "What if when jurors are unable to assess the reliability, outside council is brought in to help them assess reliability and their decisions are always based on that." Would this train of thought be correct, and that this answer is correct, not because it has to follow from the conclusion, but because all the other ACs are not supported at all.

Get the most out of your LSAT Prep Plus subscription.

Analyze and track your performance with our Testing and Analytics Package.